Đến nội dung

Hình ảnh

XUNG QUANH ĐẠI LƯỢNG $A-B, B-C, C-A$

- - - - -

  • Please log in to reply
Chưa có bài trả lời

#1
Trung Gauss

Trung Gauss

    Hạ sĩ

  • Thành viên
  • 86 Bài viết

XUNG QUANH ĐẠI LƯỢNG $A-B, B-C, C-A$
 

     Nhận xét chung: Khi đứng trước các bài toán bất đẳng thức ở dạng hoán vị hoặc đối xứng, một ý tưởng hiển nhiên là ta sẽ liên tưởng tới nhận xét: "Mọi đa thức đối xứng đều có thể biểu diễn dưới dạng đối xứng cơ sở". Cụ thể là với đa thức hoán vị vòng quanh $f(a,b,c)$ thì ta có: $$f(a,b,c)=\dfrac{1}{2}\left[ f(a,b,c)+f(c,b,a)\right] +\dfrac{1}{2}\left[ f(a, b, c)-f(c, b, a)\right]$$ Khi đó đại lượng $f(a,b,c)-f(c,b,a)$ có thể phân tích thành các đại lượng theo $a-b, b-c, c-a$. Đồng thời, khi vai trò các biến như nhau, ta hoàn toàn có thể giả sử $a=\min\{a,b,c\}$ hoặc $a=\max\{a, b, c\}$ hoặc $a\ge b\ge c$. Từ đó tìm hướng tiếp cận cụ thể cho bài toán.
 

Bài toán 1:(Trần Nam Dũng, VMO 2008) Cho $a, b, c$ là các số thực không âm đôi một phân biệt. Chứng minh rằng: $$\dfrac{1}{(b-c)^2}+\dfrac{1}{(c-a)^2}+\dfrac{1}{(a-b)^2}\ge\dfrac{4}{ab+bc+ca}$$

  Lời giải: Không giảm tính tổng quát, ta có thể giả sử $c=\min\{a,b,c\}$. Ta nhận thấy: $$(a-c)^2+(b-c)^2=(a-b)^2+2(a-c)(b-c)$$ Suy ra: $$\begin{aligned}\dfrac{1}{(b-c)^2}+\dfrac{1}{(c-a)^2}+\dfrac{1}{(a-b)^2}&=\dfrac{1}{(a-b)^2}+\dfrac{(a-c)^2+(b-c)^2}{(a-b)^2(b-c)^2}\\&=\dfrac{1}{(a-b)^2}+\dfrac{(a-b)^2}{(b-c)^2(a-c)^2}+\dfrac{2}{(a-c)(b-c)}\\&\ge\dfrac{2}{(a-c)(b-c)}+\dfrac{2}{(a-c)(b-c)}=\dfrac{4}{(a-c)(b-c)}\end{aligned}$$ Mặt khác, do $c=\min\{a, b, c\}$ nên: $$c(2a+2b-c)\ge 0\Rightarrow \dfrac{4}{(a-c)(b-c)}\ge\dfrac{4}{ab+bc+ca}$$ Từ đó ta có đpcm.
 

Cách khác:

Không mất tính tổng quát, giả sử $a>b>c\ge 0$. Đặt $x=a-b, y=b-c$ Khi đó, $x, y>0$ và: $$ab+bc+ca\ge ab=(c+y)(c+x+y)\ge y(x+y)$$ Theo đó, ta lui về chứng minh một kết quả mạnh hơn: $$\begin{aligned} &\;\;\;\;\;\;\dfrac{1}{x^2}+\dfrac{1}{y^2}+\dfrac{1}{(x+y)^2}\ge\dfrac{4}{y(x+y)}\\&\Leftrightarrow y(x+y)\left[\dfrac{1}{x^2}+\dfrac{1}{y^2}+\dfrac{1}{(x+y)^2}\right]\ge 4\\&\Leftrightarrow \dfrac{y}{x}\left( 1+\dfrac{y}{x}\right)+\dfrac{x}{y}+\dfrac{y}{x+y}\ge 3\end{aligned}$$ Đặt $t=\dfrac{x}{y}$, lúc này ta cần chứng minh: $$\dfrac{t+1}{t^2}+t+\dfrac{1}{t+1}\ge 3$$ Bất đẳng thức này tương đương với: $$\dfrac{(t^2-t-1)^2}{t^2(t+1)}\ge 0$$ Đây là một kết quả hiển nhiên. Bài toán được chứng minh

 

. Bài toán 2: Cho $a, b, c$ là các số thực phân biệt. Chứng minh rằng: $$\left[(a-b)^4+(b-c)^4+(c-a)^4\right]\left[\dfrac{1}{(a-b)^4}+\dfrac{1}{(b-c)^4}+\dfrac{1}{(c-a)^4}\right]\ge\dfrac{297}{8}$$

  Lời giải:

Không giảm tính tổng quát, ta có thể giả sử $a\ge b\ge c$. Đặt $x=a-b, y=b-c$. Khi đó ta có $x, y\ge 0$ và $c-a=-x-y$. Bất đẳng thức cần chứng minh tương đương với: $$\left[ x^4+y^4+(x+y)^4\right]\left[\dfrac{1}{x^4}+\dfrac{1}{y^4}+\dfrac{1}{(x+y)^4}\right]\ge\dfrac{297}{8}$$ Tới đây, ta có đánh giá: $$x^4+y^4\ge\dfrac{(x+y)^4}{8}$$ và $$\dfrac{1}{x^4}+\dfrac{1}{y^4}\ge\dfrac{32}{(x+y)^4}$$ Suy ra: $$\left[ x^4+y^4+(x+y)^4\right]\left[\dfrac{1}{x^4}+\dfrac{1}{y^4}+\dfrac{1}{(x+y)^4}\right]\ge\left[\dfrac{(x+y)^4}{8}+(x+y)^4\right]\left[\dfrac{32}{(x+y)^4}+\dfrac{1}{(x+y)^4}\right]=\dfrac{297}{8}$$ Phép chứng minh hoàn tất.
 

  Bài toán 3: (Trần Quốc Anh) Cho $a, b, c$ là các số thực không âm thỏa mãn $a+b+c=3$. Tìm giá trị lớn nhất của biểu thức: $$P=(a^3+b^3+c^3)(a^2-b^2)(b^2-c^2)(c^2-a^2)$$

  Lời giải:

  Do cần tìm giá trị lớn nhất của $P$ nên ta chỉ cần xét trong trường hợp $a\le b\le c$ là được. Áp dụng bất đẳng thức AM-GM, ta có đánh giá: $$\begin{aligned}(a+b+c)^3&=a^3+b^3+c^3+3(a+b)(b+c)(c+a)\\&\ge 2\sqrt{3(a^3+b^3+c^3)(a+b)(b+c)(c+a)}\end{aligned}$$ Suy ra: $$(a^3+b^3+c^3)(a+b)(b+c)(c+a)\le\dfrac{243}{4}$$ Do $a\le b\le c$ nên $$(a-b)^2\le b^2, (c-a)^2\le c^2$$. Kéo theo: $$\begin{aligned}(a-b)^2(b-c)^2(c-a)^2&\le b^2c^2(b-c)^2=\dfrac{1}{4}.2bc.2bc.(b^2-2bc+c^2)\\&\le \dfrac{1}{4}\left(\dfrac{2bc+2bc+b^2-2bc+c^2}{3}\right)^3=\dfrac{(b+c)^6}{108}\le\dfrac{(a+b+c)^6}{108}=\dfrac{27}{4}\end{aligned}$$ Suy ra: $$(a-b)(b-c)(c-a)\le \dfrac{3\sqrt{3}}{2}$$ Từ đó, ta có ngay: $$P=(a^3+b^3+c^3)(a^2-b^2)(b^2-c^2)(c^2-a^2)\le\dfrac{729\sqrt{3}}{8}$$ Dấu đẳng thức xảy ra khi và chỉ khi $(a,b,c)=(0,3-\sqrt{3},3+\sqrt{3})$ và các hoán vị tương ứng. Kết luận $\max P=\dfrac{729\sqrt{3}}{8}$.
 

  Bài toán 4: (Phan Thành Nam) Cho $a, b, c$ là các số thực không âm. Chứng minh rằng: $$(a^2+b^2+c^2)^2\ge 4(a+b+c)(a-b)(b-c)(c-a)$$

  Lời giải:

  Ta chỉ cần xét trong trường hợp $a\le b\le c$ là đủ. Sử dụng bất đẳng thức AM-GM, ta có: $$\begin{aligned}4(a+b+c)(a-b)(b-c)(c-a)&=4(a+b+c)(b-a).(c-b)(c-a)\\&\le\left[(a+b+c)(b-a)+(c-b)(c-a)\right]^2\\&=\left[b^2+c^2+(b-2c)a-a^2\right]^2\\&\le (b^2+c^2)^2\le (a^2+b^2+c^2)^2\end{aligned}$$ Phép chứng minh hoàn tất.
 

  Bài toán 5: Cho $a, b, c$ là các số thực thỏa $a^2+b^2+c^2=1$. Tìm giá trị lớn nhất và nhỏ nhất của: $$P=(a+b+c)(a-b)(b-c)(c-a)$$

  Lời giải:

  Xét $$P^2=(a+b+c)^2(a-b)^2(b-c)^2(c-a)^2$$ Đặt $p=a+b+c, q=ab+bc+ca, r=abc$. Khi đó ta có: $$(a-b)^2(b-c)^2(c-a)^2=-27r^2+2(9pq-2p^3)r+p^2q^2-4q^3$$ Suy ra: $$P^2=(1+2q)(-27r^2+2(9pq-2p^3)r+p^2q^2-4q^3)$$ Xem $P^2$ như là một tam thức bậc hai ẩn $r$. Ta tính được: $$\Delta '=(1+2q)^2\left[(9pq-2p^3)^2+27(p^2q^2-4q^3)\right]$$ Chú ý rằng $p^2-2q=1$ và sau một vài bước biến đổi cơ bản ta dễ dàng suy ra được: $$\Delta '=4(1+2q)^2(1-q)^3$$ Từ đó suy ra, $$P^2\le\dfrac{4(1+2q)^2(1-q)^3}{27}$$ Thực hiện khảo sát hàm số $$f(q)=\dfrac{4(1+2q)^2(1-q)^3}{27}$$ ta dễ dàng suy ra: $$f(q)=\dfrac{4(1+2q)^2(1-q)^3}{27}\le f\left(-\dfrac{1}{8}\right)=\dfrac{81}{512}$$. Từ đó suy ra: $$P^2\le\dfrac{81}{512}$$ hay $$-\dfrac{9\sqrt{2}}{32}\le P\le \dfrac{9\sqrt{2}}{32}$$ Từ đó suy ra:

  + $\min P=-\dfrac{9\sqrt{2}}{32}$ đạt được khi và chỉ khi $(a, b, c)=\left(\dfrac{3\sqrt{6}+2\sqrt{3}}{12}, \dfrac{\sqrt{3}}{6}, \dfrac{2\sqrt{3}-3\sqrt{6}}{12}\right)$ và các hoán vị tương ứng.

  + $\max P=\dfrac{9\sqrt{2}}{32}$ đạt được khi và chỉ khi $(a, b, c)=\left(\dfrac{2\sqrt{3}-3\sqrt{6}}{12}, \dfrac{\sqrt{3}}{6},\dfrac{3\sqrt{6}+2\sqrt{3}}{12}\right)$ và các hoán vị tương ứng.

 

   Nếu ta gọi $M$ là giá trị lớn nhất (hay nhỏ nhất) của $P$. Sau khi thuần nhất hóa, ta cần định giá trị $M$ nhỏ nhất sao cho bất đẳng thức sau đúng: $(a-b)^2(b-c)^2(c-a)^2(a+b+c)^2\ge M^2(a^2+b^2+c^2)^4$ Dĩ nhiên là lúc này ta bỏ qua điều kiện $a^2+b^2+c^2=1$. Tiếp tục biến đổi ta thu được bài toán của kì thi IMO 2006:

Bài toán:(IMO 2006) Tìm số thực $M$ nhỏ nhất sao cho với mọi $a, b, c$ thực, ta có bất đẳng thức: $$|ab(a^2-b^2)+bc(b^2-c^2)+ca(c^2-a^2)|\le M(a^2+b^2+c^2)^2$$

 

  Bài toán 6: Cho $a, b, c$ là các số không âm có tổng bằng 1. Tìm giá trị lớn nhất của: $$P=(a-b)^3+(b-c)^3+(c-a)^3$$

  Lời giải:

  Ta chú ý thấy $a-b+b-c+c-a=0$ nên dễ dàng suy ra: $$(a-b)^3+(b-c)^3+(c-a)^3=3(a-b)(b-c)(c-a)$$ Theo đó, ta cần tìm giá trị lớn nhất của $$Q=(a-b)(b-c)(c-a)$$ Không mất tính tổng quát, giả sử $a\le b\le c$. Sử dụng bất đẳng thức AM-GM, ta có: $$\begin{aligned}Q&\le\dfrac{1}{2}\left[\dfrac{(\sqrt{3}+1)(b-a)+(\sqrt{3}-1)(c-a)+(c-b)}{3}\right]^3\\&=\dfrac{1}{2}\left(\dfrac{b+c-2a}{\sqrt{3}}\right)^3=\dfrac{1}{2}\left(\dfrac{1-3a}{\sqrt{3}}\right)^3\le\dfrac{1}{2.(\sqrt{3})^3}=\dfrac{\sqrt{3}}{18}\end{aligned}$$ Suy ra: $$P=3Q\le\dfrac{\sqrt{3}}{6}$$ Dấu đẳng thức xảy ra khi $(a,b,c)=\left(0,\dfrac{3-\sqrt{3}}{6}, \dfrac{3+\sqrt{3}}{6}\right)$ và các hoán vị tương ứng. Kết luận: $\max P=\dfrac{\sqrt{3}}{6}$
 

  Bài toán 7: Cho $a, b, c$ là các số thực phân biệt có tổng bằng $1$ và thỏa $ab+bc+ca>0$. Tìm giá trị nhỏ nhất của: $$P=\dfrac{2}{|a-b|}+\dfrac{2}{|b-c|}+\dfrac{2}{|c-a|}+\dfrac{5}{\sqrt{ab+bc+ca}}$$

  Lời giải:

  Không mất tính tổng quát, giả sử $a<b<c$. Khi đó, ta có: $$P=\dfrac{2}{b-a}+\dfrac{2}{c-b}+\dfrac{2}{c-a}+\dfrac{5}{\sqrt{ab+bc+ca}}$$ Áp dụng bất đẳng thức Cauchy Schwarz, ta có: $$P\ge \dfrac{10}{c-a}+\dfrac{5}{\sqrt{ab+bc+ca}}$$ Ngoài ra, theo AM-GM ta còn có: $$\begin{aligned} 3(ab+bc+ca)-1&=3(ab+bc+ca)-(a+b+c)^2\\&=ab+bc+ca-a^2-b^2-c^2\\&=-(c-a)^2+(c-b)(b-a)\\&\le -(c-a)^2+ \dfrac{(c-b+b-a)^2}{4} =-\dfrac{3(c-a)^2}{4}\end{aligned}$$ Suy ra: $$ab+bc+ca\le \dfrac{4-3(c-a)^2}{12}$$ Từ đó suy ra: $$P\ge \dfrac{10}{c-a}+\dfrac{10\sqrt{3}}{\sqrt{4-3(c-a)^2}}$$ Đặt $x=c-a, x>0$ thế thì:$$P\ge \dfrac{10}{x}+\dfrac{10\sqrt{3}}{\sqrt{4-3x^2}}$$ Tới đây, bằng việc khảo sát hàm số $$f(x)=\dfrac{10}{x}+\dfrac{10\sqrt{3}}{\sqrt{4-3x^2}}$$ ta dễ dàng nhận thấy $$f(x)\ge 10\sqrt{6}$$. Cho nên ta có ngay: $$P\ge 10\sqrt{6}$$ Dấu đẳng thức xảy ra khi: $\begin{cases}a+b+c=1\\b-a=c-b\\c-a=\sqrt{\dfrac{2}{3}}\\ab+bc+ca>0\end{cases}\Leftrightarrow \begin{cases}a=\dfrac{1}{3}-\dfrac{1}{\sqrt{6}}\\\\b=\dfrac{1}{3}\\\\c=\dfrac{1}{3}+\dfrac{1}{\sqrt{6}}\end{cases}$

Kết luận, $\min P=10\sqrt{6}$ đạt được khi $(a,b,c)=\left(\dfrac{1}{3}-\dfrac{1}{\sqrt{6}},\dfrac{1}{3},\dfrac{1}{3}+\dfrac{1}{\sqrt{6}}\right)$ và các hoán vị tương ứng.
 

   Cuối cùng, kết thúc bài viết tôi xin đề nghị một số bài tập tự luyện:

   1. (Trần Quốc Anh) Cho $a, b, c$ là các số thực không âm. Chứng minh rằng: $$(a^2+b^2+c^2)^3\ge 27(a-b)^2(b-c)^2(c-a)^2$$

  2. Cho $a, b, c$ là các số thực dương. Chứng minh rằng: $$\sqrt[3]{(a+b)^2(b+c)^2(c+a)^2}\ge\sqrt[3]{(a-b)^2(b-c)^2(c-a)^2}+4\sqrt[3]{a^2b^2c^2}$$

  3. (IMO 1983) Cho $a, b, c$ là độ dài ba cạnh của một tam giác. Chứng minh rằng: $$a^2b(a-b)+b^2c(b-c)+c^2a(c-a)\ge 0$$

  4.(Nguyễn Văn Thạch) Cho các số thực dương $a, b, c$ đôi một phân biệt. Chứng minh rằng: $$\dfrac{1+a^2b^2}{(a-b)^2}+\dfrac{1+b^2c^2}{(b-c)^2}+\dfrac{1+c^2a^2}{(c-a)^2}\ge\dfrac{3}{2}$$

  5. (Võ Quốc Bá Cẩn) Cho $a, b, c$ là các số thực không âm có tổng bình phương bằng $1$. Tìm giá trị lớn nhất của: $$P=(a-b)(b-c)(c-a)(a+b+c)$$

Tài liệu tham khảo: 

$\left[ 1\right]$. Vẻ đẹp bất đẳng thức qua các kỳ thi Olympic - Trần Phương (chủ biên).

$\left[ 2\right]$. Bất đẳng thức hiện đại - Võ Quốc Bá Cẩn.

$\left[ 3\right]$. Một số tài liệu từ Internet.


Bài viết đã được chỉnh sửa nội dung bởi Trung Gauss: 28-01-2015 - 20:59





0 người đang xem chủ đề

0 thành viên, 0 khách, 0 thành viên ẩn danh